Recipe for embedding $m$-fold product $X_m := S^1 times … times S^1$ onto a bounded subset of...












2














I am an undergraduate student doing a first course in topology. I am stuck with a conclusion that I feel like should be relatively straightforward after proving quite some statements. The assignment is relatively long (giving a "recipe") for the final conclusion, so please bear with me.



Let $S^1$ denote the unit circle in $mathbb{R}^2$. Consider for $n geq 2$ the mapping:



$R: S^1 rightarrow M_{n+1}$ given by $(u,v) mapsto$ $ begin{bmatrix}
u & 0 & -v \
0 & I_{n-1} & 0 \
v & 0 & u
end{bmatrix}$



I have proved the following things:



$(textbf{A})$ that $R(cos (phi), sin(phi))$ defines a rotation by an angle $phi$ in the $(x_{1}, x_{n+1})$-plane.



Now, letting $H:={x in mathbb{R}^n vert x_1 >0 }$



$(textbf{B})$ that the map $f:H times S^1 mapsto mathbb{R}^{n+1}$, $(x, u, v) mapsto $
$R(u,v)begin{bmatrix}
x_{1} \
x_{2} \
vdots \
x_{n}\
0
end{bmatrix}$

is a homeomorphism from



$H times$ $S^1$ (equipped with the topology induced from $mathbb{R}^n times mathbb{R}^2$)
onto an open subset of $mathbb{R}^{n+1}$.



$(textbf{C})$ When $A subset H$ and $A times S^1 subset mathbb{R}^n times mathbb{R}^2$ are equipped with the induced topology, then $f|_{A×S^1}$ is an embedding of $A times S^1$ onto $mathbb{R}^{n+1}$.



$(textbf{D})$Finally I showed that if $A$ is bounded in $mathbb{R}^n$ that $f(A times S^1)$ is bounded in $mathbb{R}^{n+1}$.



Using these results I have to conclude the following:



For $m geq$ 1 let the $m$-fold product $X_m := S^1 times ... times S^1 subset (mathbb{R}^2)^m = mathbb{R}^{2m}$ be equipped with induced topology. Show for all $m geq 1$ there exists an embedding of $X_m$ onto a bounded subset of $mathbb{R}^{m+1}$ .



I feel like one can picture the $(m-1)$-fold product of $S^1$ to sit in $H$, as the bounded subset $A$ from part $textbf{D}$. But I do not know how to realize this since $S^1$ is the unit circle and the first coordinate of elements of $H$ is greater than $0$.



Any help would be appreciated!










share|cite|improve this question



























    2














    I am an undergraduate student doing a first course in topology. I am stuck with a conclusion that I feel like should be relatively straightforward after proving quite some statements. The assignment is relatively long (giving a "recipe") for the final conclusion, so please bear with me.



    Let $S^1$ denote the unit circle in $mathbb{R}^2$. Consider for $n geq 2$ the mapping:



    $R: S^1 rightarrow M_{n+1}$ given by $(u,v) mapsto$ $ begin{bmatrix}
    u & 0 & -v \
    0 & I_{n-1} & 0 \
    v & 0 & u
    end{bmatrix}$



    I have proved the following things:



    $(textbf{A})$ that $R(cos (phi), sin(phi))$ defines a rotation by an angle $phi$ in the $(x_{1}, x_{n+1})$-plane.



    Now, letting $H:={x in mathbb{R}^n vert x_1 >0 }$



    $(textbf{B})$ that the map $f:H times S^1 mapsto mathbb{R}^{n+1}$, $(x, u, v) mapsto $
    $R(u,v)begin{bmatrix}
    x_{1} \
    x_{2} \
    vdots \
    x_{n}\
    0
    end{bmatrix}$

    is a homeomorphism from



    $H times$ $S^1$ (equipped with the topology induced from $mathbb{R}^n times mathbb{R}^2$)
    onto an open subset of $mathbb{R}^{n+1}$.



    $(textbf{C})$ When $A subset H$ and $A times S^1 subset mathbb{R}^n times mathbb{R}^2$ are equipped with the induced topology, then $f|_{A×S^1}$ is an embedding of $A times S^1$ onto $mathbb{R}^{n+1}$.



    $(textbf{D})$Finally I showed that if $A$ is bounded in $mathbb{R}^n$ that $f(A times S^1)$ is bounded in $mathbb{R}^{n+1}$.



    Using these results I have to conclude the following:



    For $m geq$ 1 let the $m$-fold product $X_m := S^1 times ... times S^1 subset (mathbb{R}^2)^m = mathbb{R}^{2m}$ be equipped with induced topology. Show for all $m geq 1$ there exists an embedding of $X_m$ onto a bounded subset of $mathbb{R}^{m+1}$ .



    I feel like one can picture the $(m-1)$-fold product of $S^1$ to sit in $H$, as the bounded subset $A$ from part $textbf{D}$. But I do not know how to realize this since $S^1$ is the unit circle and the first coordinate of elements of $H$ is greater than $0$.



    Any help would be appreciated!










    share|cite|improve this question

























      2












      2








      2







      I am an undergraduate student doing a first course in topology. I am stuck with a conclusion that I feel like should be relatively straightforward after proving quite some statements. The assignment is relatively long (giving a "recipe") for the final conclusion, so please bear with me.



      Let $S^1$ denote the unit circle in $mathbb{R}^2$. Consider for $n geq 2$ the mapping:



      $R: S^1 rightarrow M_{n+1}$ given by $(u,v) mapsto$ $ begin{bmatrix}
      u & 0 & -v \
      0 & I_{n-1} & 0 \
      v & 0 & u
      end{bmatrix}$



      I have proved the following things:



      $(textbf{A})$ that $R(cos (phi), sin(phi))$ defines a rotation by an angle $phi$ in the $(x_{1}, x_{n+1})$-plane.



      Now, letting $H:={x in mathbb{R}^n vert x_1 >0 }$



      $(textbf{B})$ that the map $f:H times S^1 mapsto mathbb{R}^{n+1}$, $(x, u, v) mapsto $
      $R(u,v)begin{bmatrix}
      x_{1} \
      x_{2} \
      vdots \
      x_{n}\
      0
      end{bmatrix}$

      is a homeomorphism from



      $H times$ $S^1$ (equipped with the topology induced from $mathbb{R}^n times mathbb{R}^2$)
      onto an open subset of $mathbb{R}^{n+1}$.



      $(textbf{C})$ When $A subset H$ and $A times S^1 subset mathbb{R}^n times mathbb{R}^2$ are equipped with the induced topology, then $f|_{A×S^1}$ is an embedding of $A times S^1$ onto $mathbb{R}^{n+1}$.



      $(textbf{D})$Finally I showed that if $A$ is bounded in $mathbb{R}^n$ that $f(A times S^1)$ is bounded in $mathbb{R}^{n+1}$.



      Using these results I have to conclude the following:



      For $m geq$ 1 let the $m$-fold product $X_m := S^1 times ... times S^1 subset (mathbb{R}^2)^m = mathbb{R}^{2m}$ be equipped with induced topology. Show for all $m geq 1$ there exists an embedding of $X_m$ onto a bounded subset of $mathbb{R}^{m+1}$ .



      I feel like one can picture the $(m-1)$-fold product of $S^1$ to sit in $H$, as the bounded subset $A$ from part $textbf{D}$. But I do not know how to realize this since $S^1$ is the unit circle and the first coordinate of elements of $H$ is greater than $0$.



      Any help would be appreciated!










      share|cite|improve this question













      I am an undergraduate student doing a first course in topology. I am stuck with a conclusion that I feel like should be relatively straightforward after proving quite some statements. The assignment is relatively long (giving a "recipe") for the final conclusion, so please bear with me.



      Let $S^1$ denote the unit circle in $mathbb{R}^2$. Consider for $n geq 2$ the mapping:



      $R: S^1 rightarrow M_{n+1}$ given by $(u,v) mapsto$ $ begin{bmatrix}
      u & 0 & -v \
      0 & I_{n-1} & 0 \
      v & 0 & u
      end{bmatrix}$



      I have proved the following things:



      $(textbf{A})$ that $R(cos (phi), sin(phi))$ defines a rotation by an angle $phi$ in the $(x_{1}, x_{n+1})$-plane.



      Now, letting $H:={x in mathbb{R}^n vert x_1 >0 }$



      $(textbf{B})$ that the map $f:H times S^1 mapsto mathbb{R}^{n+1}$, $(x, u, v) mapsto $
      $R(u,v)begin{bmatrix}
      x_{1} \
      x_{2} \
      vdots \
      x_{n}\
      0
      end{bmatrix}$

      is a homeomorphism from



      $H times$ $S^1$ (equipped with the topology induced from $mathbb{R}^n times mathbb{R}^2$)
      onto an open subset of $mathbb{R}^{n+1}$.



      $(textbf{C})$ When $A subset H$ and $A times S^1 subset mathbb{R}^n times mathbb{R}^2$ are equipped with the induced topology, then $f|_{A×S^1}$ is an embedding of $A times S^1$ onto $mathbb{R}^{n+1}$.



      $(textbf{D})$Finally I showed that if $A$ is bounded in $mathbb{R}^n$ that $f(A times S^1)$ is bounded in $mathbb{R}^{n+1}$.



      Using these results I have to conclude the following:



      For $m geq$ 1 let the $m$-fold product $X_m := S^1 times ... times S^1 subset (mathbb{R}^2)^m = mathbb{R}^{2m}$ be equipped with induced topology. Show for all $m geq 1$ there exists an embedding of $X_m$ onto a bounded subset of $mathbb{R}^{m+1}$ .



      I feel like one can picture the $(m-1)$-fold product of $S^1$ to sit in $H$, as the bounded subset $A$ from part $textbf{D}$. But I do not know how to realize this since $S^1$ is the unit circle and the first coordinate of elements of $H$ is greater than $0$.



      Any help would be appreciated!







      general-topology metric-spaces






      share|cite|improve this question













      share|cite|improve this question











      share|cite|improve this question




      share|cite|improve this question










      asked Nov 28 '18 at 16:59









      HK4

      133




      133






















          1 Answer
          1






          active

          oldest

          votes


















          1














          We prove that $X_m$ embeds into $mathbb{R}^{m+1}$ as a bounded subset by induction on $m$.



          The base case $m=1$ is just the statement that $S^1subseteq mathbb{R}^2$ is bounded.



          Now assume inductively that $X_m$ embeds into $mathbb{R}^{m+1}$ as a bounded subset. Let $g:X_mrightarrow mathbb{R}^{m+1}$ be such an embedding.



          Because $g(X_m)$ is bounded it lies inside of some cube: $g(X_m)subseteq [a_1, b_1]times [a_2,b_2]times...times [a_{m+1}, b_{m+1}]$. Consider a new function $h:X_mrightarrow mathbb{R}^{m+1}$ given by $h(x) = g(x) + (2|a_1|,0,...,0)$. I leave it to you to show that $h$ is also an embedding and that $h(X_m)subseteq H$.



          Great. How does this give an embedding of $X_{m+1}$ into $mathbb{R}^{m+2}$? Well, now that we have $h(X_m)subseteq A$, part C can be used. Can you finish from here?






          share|cite|improve this answer





















            Your Answer





            StackExchange.ifUsing("editor", function () {
            return StackExchange.using("mathjaxEditing", function () {
            StackExchange.MarkdownEditor.creationCallbacks.add(function (editor, postfix) {
            StackExchange.mathjaxEditing.prepareWmdForMathJax(editor, postfix, [["$", "$"], ["\\(","\\)"]]);
            });
            });
            }, "mathjax-editing");

            StackExchange.ready(function() {
            var channelOptions = {
            tags: "".split(" "),
            id: "69"
            };
            initTagRenderer("".split(" "), "".split(" "), channelOptions);

            StackExchange.using("externalEditor", function() {
            // Have to fire editor after snippets, if snippets enabled
            if (StackExchange.settings.snippets.snippetsEnabled) {
            StackExchange.using("snippets", function() {
            createEditor();
            });
            }
            else {
            createEditor();
            }
            });

            function createEditor() {
            StackExchange.prepareEditor({
            heartbeatType: 'answer',
            autoActivateHeartbeat: false,
            convertImagesToLinks: true,
            noModals: true,
            showLowRepImageUploadWarning: true,
            reputationToPostImages: 10,
            bindNavPrevention: true,
            postfix: "",
            imageUploader: {
            brandingHtml: "Powered by u003ca class="icon-imgur-white" href="https://imgur.com/"u003eu003c/au003e",
            contentPolicyHtml: "User contributions licensed under u003ca href="https://creativecommons.org/licenses/by-sa/3.0/"u003ecc by-sa 3.0 with attribution requiredu003c/au003e u003ca href="https://stackoverflow.com/legal/content-policy"u003e(content policy)u003c/au003e",
            allowUrls: true
            },
            noCode: true, onDemand: true,
            discardSelector: ".discard-answer"
            ,immediatelyShowMarkdownHelp:true
            });


            }
            });














            draft saved

            draft discarded


















            StackExchange.ready(
            function () {
            StackExchange.openid.initPostLogin('.new-post-login', 'https%3a%2f%2fmath.stackexchange.com%2fquestions%2f3017388%2frecipe-for-embedding-m-fold-product-x-m-s1-times-times-s1-ont%23new-answer', 'question_page');
            }
            );

            Post as a guest















            Required, but never shown

























            1 Answer
            1






            active

            oldest

            votes








            1 Answer
            1






            active

            oldest

            votes









            active

            oldest

            votes






            active

            oldest

            votes









            1














            We prove that $X_m$ embeds into $mathbb{R}^{m+1}$ as a bounded subset by induction on $m$.



            The base case $m=1$ is just the statement that $S^1subseteq mathbb{R}^2$ is bounded.



            Now assume inductively that $X_m$ embeds into $mathbb{R}^{m+1}$ as a bounded subset. Let $g:X_mrightarrow mathbb{R}^{m+1}$ be such an embedding.



            Because $g(X_m)$ is bounded it lies inside of some cube: $g(X_m)subseteq [a_1, b_1]times [a_2,b_2]times...times [a_{m+1}, b_{m+1}]$. Consider a new function $h:X_mrightarrow mathbb{R}^{m+1}$ given by $h(x) = g(x) + (2|a_1|,0,...,0)$. I leave it to you to show that $h$ is also an embedding and that $h(X_m)subseteq H$.



            Great. How does this give an embedding of $X_{m+1}$ into $mathbb{R}^{m+2}$? Well, now that we have $h(X_m)subseteq A$, part C can be used. Can you finish from here?






            share|cite|improve this answer


























              1














              We prove that $X_m$ embeds into $mathbb{R}^{m+1}$ as a bounded subset by induction on $m$.



              The base case $m=1$ is just the statement that $S^1subseteq mathbb{R}^2$ is bounded.



              Now assume inductively that $X_m$ embeds into $mathbb{R}^{m+1}$ as a bounded subset. Let $g:X_mrightarrow mathbb{R}^{m+1}$ be such an embedding.



              Because $g(X_m)$ is bounded it lies inside of some cube: $g(X_m)subseteq [a_1, b_1]times [a_2,b_2]times...times [a_{m+1}, b_{m+1}]$. Consider a new function $h:X_mrightarrow mathbb{R}^{m+1}$ given by $h(x) = g(x) + (2|a_1|,0,...,0)$. I leave it to you to show that $h$ is also an embedding and that $h(X_m)subseteq H$.



              Great. How does this give an embedding of $X_{m+1}$ into $mathbb{R}^{m+2}$? Well, now that we have $h(X_m)subseteq A$, part C can be used. Can you finish from here?






              share|cite|improve this answer
























                1












                1








                1






                We prove that $X_m$ embeds into $mathbb{R}^{m+1}$ as a bounded subset by induction on $m$.



                The base case $m=1$ is just the statement that $S^1subseteq mathbb{R}^2$ is bounded.



                Now assume inductively that $X_m$ embeds into $mathbb{R}^{m+1}$ as a bounded subset. Let $g:X_mrightarrow mathbb{R}^{m+1}$ be such an embedding.



                Because $g(X_m)$ is bounded it lies inside of some cube: $g(X_m)subseteq [a_1, b_1]times [a_2,b_2]times...times [a_{m+1}, b_{m+1}]$. Consider a new function $h:X_mrightarrow mathbb{R}^{m+1}$ given by $h(x) = g(x) + (2|a_1|,0,...,0)$. I leave it to you to show that $h$ is also an embedding and that $h(X_m)subseteq H$.



                Great. How does this give an embedding of $X_{m+1}$ into $mathbb{R}^{m+2}$? Well, now that we have $h(X_m)subseteq A$, part C can be used. Can you finish from here?






                share|cite|improve this answer












                We prove that $X_m$ embeds into $mathbb{R}^{m+1}$ as a bounded subset by induction on $m$.



                The base case $m=1$ is just the statement that $S^1subseteq mathbb{R}^2$ is bounded.



                Now assume inductively that $X_m$ embeds into $mathbb{R}^{m+1}$ as a bounded subset. Let $g:X_mrightarrow mathbb{R}^{m+1}$ be such an embedding.



                Because $g(X_m)$ is bounded it lies inside of some cube: $g(X_m)subseteq [a_1, b_1]times [a_2,b_2]times...times [a_{m+1}, b_{m+1}]$. Consider a new function $h:X_mrightarrow mathbb{R}^{m+1}$ given by $h(x) = g(x) + (2|a_1|,0,...,0)$. I leave it to you to show that $h$ is also an embedding and that $h(X_m)subseteq H$.



                Great. How does this give an embedding of $X_{m+1}$ into $mathbb{R}^{m+2}$? Well, now that we have $h(X_m)subseteq A$, part C can be used. Can you finish from here?







                share|cite|improve this answer












                share|cite|improve this answer



                share|cite|improve this answer










                answered Nov 28 '18 at 17:36









                Jason DeVito

                30.7k475135




                30.7k475135






























                    draft saved

                    draft discarded




















































                    Thanks for contributing an answer to Mathematics Stack Exchange!


                    • Please be sure to answer the question. Provide details and share your research!

                    But avoid



                    • Asking for help, clarification, or responding to other answers.

                    • Making statements based on opinion; back them up with references or personal experience.


                    Use MathJax to format equations. MathJax reference.


                    To learn more, see our tips on writing great answers.





                    Some of your past answers have not been well-received, and you're in danger of being blocked from answering.


                    Please pay close attention to the following guidance:


                    • Please be sure to answer the question. Provide details and share your research!

                    But avoid



                    • Asking for help, clarification, or responding to other answers.

                    • Making statements based on opinion; back them up with references or personal experience.


                    To learn more, see our tips on writing great answers.




                    draft saved


                    draft discarded














                    StackExchange.ready(
                    function () {
                    StackExchange.openid.initPostLogin('.new-post-login', 'https%3a%2f%2fmath.stackexchange.com%2fquestions%2f3017388%2frecipe-for-embedding-m-fold-product-x-m-s1-times-times-s1-ont%23new-answer', 'question_page');
                    }
                    );

                    Post as a guest















                    Required, but never shown





















































                    Required, but never shown














                    Required, but never shown












                    Required, but never shown







                    Required, but never shown

































                    Required, but never shown














                    Required, but never shown












                    Required, but never shown







                    Required, but never shown







                    Popular posts from this blog

                    Bundesstraße 106

                    Le Mesnil-Réaume

                    Ida-Boy-Ed-Garten